subject
Business, 19.10.2021 02:30 drastipatel18

7. Assuming the stock split mentioned above, prepare any journal entry that should be made. 8. Disregard the stock split (assumed above). Assume instead that a 10 percent stock dividend was declared (after treasury stock repurchase) when the market price of the common stock was $21. Prepare any journal entry that should be made.

ansver
Answers: 2

Another question on Business

question
Business, 21.06.2019 13:20
If the supply of aisle seats and middle seats on an airplane is the same, but the demand for aisle seats is greater than the demand for middle seats, then the equilibrium price of aisle seats will be less than the equilibrium price of middle seats. true false
Answers: 2
question
Business, 22.06.2019 03:50
Suppose that a worker in agland can produce either 10 units of organic grain or 2 units of incense per year, and a worker in zenland can produce either 5 units of organic grain or 15 units of incense per year. there are 20 workers in agland and 10 workers in zenland. currently the two countries do not trade. agland produces and consumes 100 units of grain and 20 units of incense per year. zenland produces and consumes 50 units of grain and no incense per year. if each country made the decision to specialize in producing the good in which it has a comparative advantage, then the combined yearly output of the two countries would increase by a. 30 units of grain and 100 units of incense. b. 30 units of grain and 150 units of incense. c. 50 units of grain and 90 units of incense. d. 50 units of grain and 130 units of ince
Answers: 1
question
Business, 22.06.2019 15:00
Portia grant is an employee who is paid monthly. for the month of january of the current year, she earned a total of $8,388. the fica tax for social security is 6.2% of the first $118,500 earned each calendar year and the fica tax rate for medicare is 1.45% of all earnings. the futa tax rate of 0.6% and the suta tax rate of 5.4% are applied to the first $7,000 of an employee's pay. the amount of federal income tax withheld from her earnings was $1,391.77. what is the total amount of taxes withheld from the portia's earnings?
Answers: 2
question
Business, 22.06.2019 22:40
When immigration adds to the size of the domestic labor pool, which of the following is likely to occur? a. wages decrease. b. productivity increases. c. consumption decreases. d. minimum wage increases.
Answers: 1
You know the right answer?
7. Assuming the stock split mentioned above, prepare any journal entry that should be made. 8. Disre...
Questions
question
Mathematics, 21.08.2019 23:30
question
Mathematics, 21.08.2019 23:30
Questions on the website: 13722363